Real Analysis - Continuity











up vote
1
down vote

favorite












a. Give an example of a function defined everywhere on the interval $[0,1]$, which does not achieve its maximum.



b. Give an example of a function defined on $mathbb{R}$, that is nowhere continuous.



c. Give an example of a continuous function defined on a bounded set, which is not uniformly continuous.



My answers are




  1. $f(x) = x^2$


  2. $f(x) = {1$, if $x$ is rational; $-1$, if $x$ is irrational$}$

  3. $f(x) = 1/x$


My workings are attached. Please help verify if the working is correct.Solutions










share|cite|improve this question







New contributor




Ty Johnson is a new contributor to this site. Take care in asking for clarification, commenting, and answering.
Check out our Code of Conduct.
















  • 2




    2. is correct. For 3, you need to specify the bounded domain. Your answer to 1. is incorrect, as the function certainly achieves its maximum on the interval $[0,1]$.
    – Ted Shifrin
    1 hour ago















up vote
1
down vote

favorite












a. Give an example of a function defined everywhere on the interval $[0,1]$, which does not achieve its maximum.



b. Give an example of a function defined on $mathbb{R}$, that is nowhere continuous.



c. Give an example of a continuous function defined on a bounded set, which is not uniformly continuous.



My answers are




  1. $f(x) = x^2$


  2. $f(x) = {1$, if $x$ is rational; $-1$, if $x$ is irrational$}$

  3. $f(x) = 1/x$


My workings are attached. Please help verify if the working is correct.Solutions










share|cite|improve this question







New contributor




Ty Johnson is a new contributor to this site. Take care in asking for clarification, commenting, and answering.
Check out our Code of Conduct.
















  • 2




    2. is correct. For 3, you need to specify the bounded domain. Your answer to 1. is incorrect, as the function certainly achieves its maximum on the interval $[0,1]$.
    – Ted Shifrin
    1 hour ago













up vote
1
down vote

favorite









up vote
1
down vote

favorite











a. Give an example of a function defined everywhere on the interval $[0,1]$, which does not achieve its maximum.



b. Give an example of a function defined on $mathbb{R}$, that is nowhere continuous.



c. Give an example of a continuous function defined on a bounded set, which is not uniformly continuous.



My answers are




  1. $f(x) = x^2$


  2. $f(x) = {1$, if $x$ is rational; $-1$, if $x$ is irrational$}$

  3. $f(x) = 1/x$


My workings are attached. Please help verify if the working is correct.Solutions










share|cite|improve this question







New contributor




Ty Johnson is a new contributor to this site. Take care in asking for clarification, commenting, and answering.
Check out our Code of Conduct.











a. Give an example of a function defined everywhere on the interval $[0,1]$, which does not achieve its maximum.



b. Give an example of a function defined on $mathbb{R}$, that is nowhere continuous.



c. Give an example of a continuous function defined on a bounded set, which is not uniformly continuous.



My answers are




  1. $f(x) = x^2$


  2. $f(x) = {1$, if $x$ is rational; $-1$, if $x$ is irrational$}$

  3. $f(x) = 1/x$


My workings are attached. Please help verify if the working is correct.Solutions







real-analysis continuity maxima-minima uniform-continuity






share|cite|improve this question







New contributor




Ty Johnson is a new contributor to this site. Take care in asking for clarification, commenting, and answering.
Check out our Code of Conduct.











share|cite|improve this question







New contributor




Ty Johnson is a new contributor to this site. Take care in asking for clarification, commenting, and answering.
Check out our Code of Conduct.









share|cite|improve this question




share|cite|improve this question






New contributor




Ty Johnson is a new contributor to this site. Take care in asking for clarification, commenting, and answering.
Check out our Code of Conduct.









asked 1 hour ago









Ty Johnson

71




71




New contributor




Ty Johnson is a new contributor to this site. Take care in asking for clarification, commenting, and answering.
Check out our Code of Conduct.





New contributor





Ty Johnson is a new contributor to this site. Take care in asking for clarification, commenting, and answering.
Check out our Code of Conduct.






Ty Johnson is a new contributor to this site. Take care in asking for clarification, commenting, and answering.
Check out our Code of Conduct.








  • 2




    2. is correct. For 3, you need to specify the bounded domain. Your answer to 1. is incorrect, as the function certainly achieves its maximum on the interval $[0,1]$.
    – Ted Shifrin
    1 hour ago














  • 2




    2. is correct. For 3, you need to specify the bounded domain. Your answer to 1. is incorrect, as the function certainly achieves its maximum on the interval $[0,1]$.
    – Ted Shifrin
    1 hour ago








2




2




2. is correct. For 3, you need to specify the bounded domain. Your answer to 1. is incorrect, as the function certainly achieves its maximum on the interval $[0,1]$.
– Ted Shifrin
1 hour ago




2. is correct. For 3, you need to specify the bounded domain. Your answer to 1. is incorrect, as the function certainly achieves its maximum on the interval $[0,1]$.
– Ted Shifrin
1 hour ago










3 Answers
3






active

oldest

votes

















up vote
4
down vote













$a)$ Consider $f(x)$ on $[0,1]$ such that $f(0) = 0, f(1) = 0, f(x) = dfrac{1}{x}, 0 <x < 1$.



$b)$ The function you had is a good one.



$c)$ Consider $f(x)$ on $(0,1)$ and $f(x) = dfrac{1}{x}$.






share|cite|improve this answer




























    up vote
    2
    down vote














    1. Wrong: the maximum is $1$, which is $f(1)$.

    2. Right.

    3. Your answer is incomplete, at best, since you did not state what is the domain of your function.






    share|cite|improve this answer

















    • 2




      Did this contribute anything beyond what I'd posted in my comment several minutes ahead of you? (I did not downvote, but ...)
      – Ted Shifrin
      53 mins ago






    • 1




      @TedShifrin Those “several minutes” consist only of two minutes and, of course, when I started to type, your comment had not appeared yet. Why did you post it as a comment instead of posting it as an answer?
      – José Carlos Santos
      51 mins ago










    • One of the take aways from this, JoséCarlosSantos is not so much why Ted Shifrin didn't post his comment as an answer, but why did you post this comment of yours as an answer, when it is really a comment, and can really be wrapped up in a comment.
      – amWhy
      43 mins ago








    • 1




      @amWhy I posted it as an answer because it is an answer.
      – José Carlos Santos
      39 mins ago


















    up vote
    1
    down vote













    1)



    Extreme value theorem says that every continuous function over a closed interval has a maximum.



    You need a function that is not continuous.



    $f(x) = begin{cases} f(x) = x & x<1\f(x) = 0 & x=1 end{cases}$






    share|cite|improve this answer





















      Your Answer





      StackExchange.ifUsing("editor", function () {
      return StackExchange.using("mathjaxEditing", function () {
      StackExchange.MarkdownEditor.creationCallbacks.add(function (editor, postfix) {
      StackExchange.mathjaxEditing.prepareWmdForMathJax(editor, postfix, [["$", "$"], ["\\(","\\)"]]);
      });
      });
      }, "mathjax-editing");

      StackExchange.ready(function() {
      var channelOptions = {
      tags: "".split(" "),
      id: "69"
      };
      initTagRenderer("".split(" "), "".split(" "), channelOptions);

      StackExchange.using("externalEditor", function() {
      // Have to fire editor after snippets, if snippets enabled
      if (StackExchange.settings.snippets.snippetsEnabled) {
      StackExchange.using("snippets", function() {
      createEditor();
      });
      }
      else {
      createEditor();
      }
      });

      function createEditor() {
      StackExchange.prepareEditor({
      heartbeatType: 'answer',
      convertImagesToLinks: true,
      noModals: true,
      showLowRepImageUploadWarning: true,
      reputationToPostImages: 10,
      bindNavPrevention: true,
      postfix: "",
      imageUploader: {
      brandingHtml: "Powered by u003ca class="icon-imgur-white" href="https://imgur.com/"u003eu003c/au003e",
      contentPolicyHtml: "User contributions licensed under u003ca href="https://creativecommons.org/licenses/by-sa/3.0/"u003ecc by-sa 3.0 with attribution requiredu003c/au003e u003ca href="https://stackoverflow.com/legal/content-policy"u003e(content policy)u003c/au003e",
      allowUrls: true
      },
      noCode: true, onDemand: true,
      discardSelector: ".discard-answer"
      ,immediatelyShowMarkdownHelp:true
      });


      }
      });






      Ty Johnson is a new contributor. Be nice, and check out our Code of Conduct.










      draft saved

      draft discarded


















      StackExchange.ready(
      function () {
      StackExchange.openid.initPostLogin('.new-post-login', 'https%3a%2f%2fmath.stackexchange.com%2fquestions%2f3021931%2freal-analysis-continuity%23new-answer', 'question_page');
      }
      );

      Post as a guest















      Required, but never shown

























      3 Answers
      3






      active

      oldest

      votes








      3 Answers
      3






      active

      oldest

      votes









      active

      oldest

      votes






      active

      oldest

      votes








      up vote
      4
      down vote













      $a)$ Consider $f(x)$ on $[0,1]$ such that $f(0) = 0, f(1) = 0, f(x) = dfrac{1}{x}, 0 <x < 1$.



      $b)$ The function you had is a good one.



      $c)$ Consider $f(x)$ on $(0,1)$ and $f(x) = dfrac{1}{x}$.






      share|cite|improve this answer

























        up vote
        4
        down vote













        $a)$ Consider $f(x)$ on $[0,1]$ such that $f(0) = 0, f(1) = 0, f(x) = dfrac{1}{x}, 0 <x < 1$.



        $b)$ The function you had is a good one.



        $c)$ Consider $f(x)$ on $(0,1)$ and $f(x) = dfrac{1}{x}$.






        share|cite|improve this answer























          up vote
          4
          down vote










          up vote
          4
          down vote









          $a)$ Consider $f(x)$ on $[0,1]$ such that $f(0) = 0, f(1) = 0, f(x) = dfrac{1}{x}, 0 <x < 1$.



          $b)$ The function you had is a good one.



          $c)$ Consider $f(x)$ on $(0,1)$ and $f(x) = dfrac{1}{x}$.






          share|cite|improve this answer












          $a)$ Consider $f(x)$ on $[0,1]$ such that $f(0) = 0, f(1) = 0, f(x) = dfrac{1}{x}, 0 <x < 1$.



          $b)$ The function you had is a good one.



          $c)$ Consider $f(x)$ on $(0,1)$ and $f(x) = dfrac{1}{x}$.







          share|cite|improve this answer












          share|cite|improve this answer



          share|cite|improve this answer










          answered 54 mins ago









          DeepSea

          70.5k54487




          70.5k54487






















              up vote
              2
              down vote














              1. Wrong: the maximum is $1$, which is $f(1)$.

              2. Right.

              3. Your answer is incomplete, at best, since you did not state what is the domain of your function.






              share|cite|improve this answer

















              • 2




                Did this contribute anything beyond what I'd posted in my comment several minutes ahead of you? (I did not downvote, but ...)
                – Ted Shifrin
                53 mins ago






              • 1




                @TedShifrin Those “several minutes” consist only of two minutes and, of course, when I started to type, your comment had not appeared yet. Why did you post it as a comment instead of posting it as an answer?
                – José Carlos Santos
                51 mins ago










              • One of the take aways from this, JoséCarlosSantos is not so much why Ted Shifrin didn't post his comment as an answer, but why did you post this comment of yours as an answer, when it is really a comment, and can really be wrapped up in a comment.
                – amWhy
                43 mins ago








              • 1




                @amWhy I posted it as an answer because it is an answer.
                – José Carlos Santos
                39 mins ago















              up vote
              2
              down vote














              1. Wrong: the maximum is $1$, which is $f(1)$.

              2. Right.

              3. Your answer is incomplete, at best, since you did not state what is the domain of your function.






              share|cite|improve this answer

















              • 2




                Did this contribute anything beyond what I'd posted in my comment several minutes ahead of you? (I did not downvote, but ...)
                – Ted Shifrin
                53 mins ago






              • 1




                @TedShifrin Those “several minutes” consist only of two minutes and, of course, when I started to type, your comment had not appeared yet. Why did you post it as a comment instead of posting it as an answer?
                – José Carlos Santos
                51 mins ago










              • One of the take aways from this, JoséCarlosSantos is not so much why Ted Shifrin didn't post his comment as an answer, but why did you post this comment of yours as an answer, when it is really a comment, and can really be wrapped up in a comment.
                – amWhy
                43 mins ago








              • 1




                @amWhy I posted it as an answer because it is an answer.
                – José Carlos Santos
                39 mins ago













              up vote
              2
              down vote










              up vote
              2
              down vote










              1. Wrong: the maximum is $1$, which is $f(1)$.

              2. Right.

              3. Your answer is incomplete, at best, since you did not state what is the domain of your function.






              share|cite|improve this answer













              1. Wrong: the maximum is $1$, which is $f(1)$.

              2. Right.

              3. Your answer is incomplete, at best, since you did not state what is the domain of your function.







              share|cite|improve this answer












              share|cite|improve this answer



              share|cite|improve this answer










              answered 1 hour ago









              José Carlos Santos

              143k20112208




              143k20112208








              • 2




                Did this contribute anything beyond what I'd posted in my comment several minutes ahead of you? (I did not downvote, but ...)
                – Ted Shifrin
                53 mins ago






              • 1




                @TedShifrin Those “several minutes” consist only of two minutes and, of course, when I started to type, your comment had not appeared yet. Why did you post it as a comment instead of posting it as an answer?
                – José Carlos Santos
                51 mins ago










              • One of the take aways from this, JoséCarlosSantos is not so much why Ted Shifrin didn't post his comment as an answer, but why did you post this comment of yours as an answer, when it is really a comment, and can really be wrapped up in a comment.
                – amWhy
                43 mins ago








              • 1




                @amWhy I posted it as an answer because it is an answer.
                – José Carlos Santos
                39 mins ago














              • 2




                Did this contribute anything beyond what I'd posted in my comment several minutes ahead of you? (I did not downvote, but ...)
                – Ted Shifrin
                53 mins ago






              • 1




                @TedShifrin Those “several minutes” consist only of two minutes and, of course, when I started to type, your comment had not appeared yet. Why did you post it as a comment instead of posting it as an answer?
                – José Carlos Santos
                51 mins ago










              • One of the take aways from this, JoséCarlosSantos is not so much why Ted Shifrin didn't post his comment as an answer, but why did you post this comment of yours as an answer, when it is really a comment, and can really be wrapped up in a comment.
                – amWhy
                43 mins ago








              • 1




                @amWhy I posted it as an answer because it is an answer.
                – José Carlos Santos
                39 mins ago








              2




              2




              Did this contribute anything beyond what I'd posted in my comment several minutes ahead of you? (I did not downvote, but ...)
              – Ted Shifrin
              53 mins ago




              Did this contribute anything beyond what I'd posted in my comment several minutes ahead of you? (I did not downvote, but ...)
              – Ted Shifrin
              53 mins ago




              1




              1




              @TedShifrin Those “several minutes” consist only of two minutes and, of course, when I started to type, your comment had not appeared yet. Why did you post it as a comment instead of posting it as an answer?
              – José Carlos Santos
              51 mins ago




              @TedShifrin Those “several minutes” consist only of two minutes and, of course, when I started to type, your comment had not appeared yet. Why did you post it as a comment instead of posting it as an answer?
              – José Carlos Santos
              51 mins ago












              One of the take aways from this, JoséCarlosSantos is not so much why Ted Shifrin didn't post his comment as an answer, but why did you post this comment of yours as an answer, when it is really a comment, and can really be wrapped up in a comment.
              – amWhy
              43 mins ago






              One of the take aways from this, JoséCarlosSantos is not so much why Ted Shifrin didn't post his comment as an answer, but why did you post this comment of yours as an answer, when it is really a comment, and can really be wrapped up in a comment.
              – amWhy
              43 mins ago






              1




              1




              @amWhy I posted it as an answer because it is an answer.
              – José Carlos Santos
              39 mins ago




              @amWhy I posted it as an answer because it is an answer.
              – José Carlos Santos
              39 mins ago










              up vote
              1
              down vote













              1)



              Extreme value theorem says that every continuous function over a closed interval has a maximum.



              You need a function that is not continuous.



              $f(x) = begin{cases} f(x) = x & x<1\f(x) = 0 & x=1 end{cases}$






              share|cite|improve this answer

























                up vote
                1
                down vote













                1)



                Extreme value theorem says that every continuous function over a closed interval has a maximum.



                You need a function that is not continuous.



                $f(x) = begin{cases} f(x) = x & x<1\f(x) = 0 & x=1 end{cases}$






                share|cite|improve this answer























                  up vote
                  1
                  down vote










                  up vote
                  1
                  down vote









                  1)



                  Extreme value theorem says that every continuous function over a closed interval has a maximum.



                  You need a function that is not continuous.



                  $f(x) = begin{cases} f(x) = x & x<1\f(x) = 0 & x=1 end{cases}$






                  share|cite|improve this answer












                  1)



                  Extreme value theorem says that every continuous function over a closed interval has a maximum.



                  You need a function that is not continuous.



                  $f(x) = begin{cases} f(x) = x & x<1\f(x) = 0 & x=1 end{cases}$







                  share|cite|improve this answer












                  share|cite|improve this answer



                  share|cite|improve this answer










                  answered 18 mins ago









                  Doug M

                  43k31752




                  43k31752






















                      Ty Johnson is a new contributor. Be nice, and check out our Code of Conduct.










                      draft saved

                      draft discarded


















                      Ty Johnson is a new contributor. Be nice, and check out our Code of Conduct.













                      Ty Johnson is a new contributor. Be nice, and check out our Code of Conduct.












                      Ty Johnson is a new contributor. Be nice, and check out our Code of Conduct.
















                      Thanks for contributing an answer to Mathematics Stack Exchange!


                      • Please be sure to answer the question. Provide details and share your research!

                      But avoid



                      • Asking for help, clarification, or responding to other answers.

                      • Making statements based on opinion; back them up with references or personal experience.


                      Use MathJax to format equations. MathJax reference.


                      To learn more, see our tips on writing great answers.





                      Some of your past answers have not been well-received, and you're in danger of being blocked from answering.


                      Please pay close attention to the following guidance:


                      • Please be sure to answer the question. Provide details and share your research!

                      But avoid



                      • Asking for help, clarification, or responding to other answers.

                      • Making statements based on opinion; back them up with references or personal experience.


                      To learn more, see our tips on writing great answers.




                      draft saved


                      draft discarded














                      StackExchange.ready(
                      function () {
                      StackExchange.openid.initPostLogin('.new-post-login', 'https%3a%2f%2fmath.stackexchange.com%2fquestions%2f3021931%2freal-analysis-continuity%23new-answer', 'question_page');
                      }
                      );

                      Post as a guest















                      Required, but never shown





















































                      Required, but never shown














                      Required, but never shown












                      Required, but never shown







                      Required, but never shown

































                      Required, but never shown














                      Required, but never shown












                      Required, but never shown







                      Required, but never shown







                      Popular posts from this blog

                      flock() on closed filehandle LOCK_FILE at /usr/bin/apt-mirror

                      Mangá

                       ⁒  ․,‪⁊‑⁙ ⁖, ⁇‒※‌, †,⁖‗‌⁝    ‾‸⁘,‖⁔⁣,⁂‾
”‑,‥–,‬ ,⁀‹⁋‴⁑ ‒ ,‴⁋”‼ ⁨,‷⁔„ ‰′,‐‚ ‥‡‎“‷⁃⁨⁅⁣,⁔
⁇‘⁔⁡⁏⁌⁡‿‶‏⁨ ⁣⁕⁖⁨⁩⁥‽⁀  ‴‬⁜‟ ⁃‣‧⁕‮ …‍⁨‴ ⁩,⁚⁖‫ ,‵ ⁀,‮⁝‣‣ ⁑  ⁂– ․, ‾‽ ‏⁁“⁗‸ ‾… ‹‡⁌⁎‸‘ ‡⁏⁌‪ ‵⁛ ‎⁨ ―⁦⁤⁄⁕